PT34.S2.Q02 - having an efficient, attractive subway system

civnetncivnetn Free Trial Member
edited July 2016 in Logical Reasoning 148 karma
I understand that E is the correct answer choice, but since this is a sufficient assumption question, isn't E describing a necessary condition? Isn't it too strong? It says New subway cars are REQUIRED. But they're not required. They're sufficient to cause an efficient, attractive subway, which in turn is sufficient to product good economic sense, but neither new subway cars NOR having an efficient and attractive subway system is NECESSARY for good economic sense.
https://7sage.com/lsat_explanations/lsat-34-section-2-question-02/

Comments

  • [Deleted User][Deleted User] Free Trial
    edited July 2016 54 karma
    The user and all related content has been deleted.
  • civnetncivnetn Free Trial Member
    148 karma
    Geez. Thanks Sean. Isn't it weird how you can get so caught up in a problem and then come back to it and wonder why in the HELL you had a problem with it?

    Thanks man!
  • [Deleted User][Deleted User] Free Trial
    54 karma
    The user and all related content has been deleted.
Sign In or Register to comment.